LSAT and Law School Admissions Forum

Get expert LSAT preparation and law school admissions advice from PowerScore Test Preparation.

User avatar
 Dave Killoran
PowerScore Staff
  • PowerScore Staff
  • Posts: 5853
  • Joined: Mar 25, 2011
|
#88198
Complete Question Explanation
(The complete setup for this game can be found here: lsat/viewtopic.php?f=171&p=88197#p88197)

The correct answer choice is (B)

Make sure to read the question stem carefully—the stem specifies that the order of the two items is from the bottom up, meaning that the first listed item is the first layer, and the second item is the second layer.

Answer choice (A) is incorrect because from the second rule M must be immediately above L.

Answer choice (B) is the correct answer. As shown in the hypothetical that solved question #16, V can be first and R can be second.

Answer choice (C) is incorrect because from the second rule L is immediately below M, and thus M can never be first.

Answer choice (D) is incorrect because from the second rule M must be immediately above L.

Answer choice (E) is incorrect because from the first rule R and S can never be consecutive layers.

Get the most out of your LSAT Prep Plus subscription.

Analyze and track your performance with our Testing and Analytics Package.